I was doing the the following problem- Prove that\( \frac { \sqrt { a+b+c } +\sqrt { a } }{ b+c } +\frac { \sqrt { a+b+c } +\sqrt { b } }{ c+a } +\frac { \sqrt { a+b+c } +\sqrt { c } }{ a+b } \ge \frac { 9+3\sqrt { 3 } }{ 2\sqrt { a+b+c } } \).
I normalized this to a+b+c=1 and simplified to get-
1−a1+1−b1+1−c1≥29+33
By Titu's lemma, ⇒1−a1+1−b1+1−c1≥3−(a+b+c)9
However the RHS is maximized when a+b+c is maximized which is at 3. ⇒1−a1+1−b1+1−c1≥3−(a+b+c)9≤3−39=29+33
This discussion board is a place to discuss our Daily Challenges and the math and science
related to those challenges. Explanations are more than just a solution — they should
explain the steps and thinking strategies that you used to obtain the solution. Comments
should further the discussion of math and science.
When posting on Brilliant:
Use the emojis to react to an explanation, whether you're congratulating a job well done , or just really confused .
Ask specific questions about the challenge or the steps in somebody's explanation. Well-posed questions can add a lot to the discussion, but posting "I don't understand!" doesn't help anyone.
Try to contribute something new to the discussion, whether it is an extension, generalization or other idea related to the challenge.
Stay on topic — we're all here to learn more about math and science, not to hear about your favorite get-rich-quick scheme or current world events.
Markdown
Appears as
*italics* or _italics_
italics
**bold** or __bold__
bold
- bulleted - list
bulleted
list
1. numbered 2. list
numbered
list
Note: you must add a full line of space before and after lists for them to show up correctly
The second last line should be;
"RHS is maximised when √a+ √b+ √c is 'minimised' " as the whole fraction is maximised when its denominator is minimised.
@Priyanshu Mishra
–
A lot of it was luck (including the level of the paper).
Regardless I found Challenge and Thrill of Pre-College Mathematics and 'Mathematical Circles' to be good at the basic level. Once you have (sufficiently) completed them go for Problem Solving Strategies(Arthur Engel) and books by Titu Andreescu. And keep trying Olympiad questions all the time.
Please help me in solving this question ( KVS JMO 2015)
A polynomial f(x) with rational coefficients leaves remainder 15, when divided by x−3 and remainder 2x+1, when divided by (x−1)2. Find the remainder when f(x) is divided by (x−3)(x−1)2.
@Priyanshu Mishra
–
I took a cubic which left a remainder 2x+1 when divided by (x−1)2 , 2x3−4x2+4x+1=(x−1)2(x+2)+2x+1. It left a remainder 31 when divided by x−3.
Note that (x−1)2 leaves a remainder of 4 when divided by x−3.
Thus, 2x3−4x2+4x+1−4(x−1)2=2x3−8x2+12x−3 leaves a remainder of 31−4(4)=15. Hence we find the remainder of 2x3−8x2+12x−3 when divided by (x−1)2(x−3).
@Siddharth G
–
Got a better solution from my friend-
Clearly, f(1)=3 and f(3)=15.
Then, f(x)=q1(x)(x−3)(x−1)2+r1(x), where r1(x)=ax2+bx+c.
Dividing by (x−1)2 and equating remainders, 2x+1=0+(ax2+bx+c−a(x−1)2) [Multiplied by a to remove the coeff. of x2] ⇒2x+1=(2a−b)x+(c−a)⇒2=2a−b,1=c−a
Solving with f(1)=3=a+b+c−(1)
(a=2, b=-2, c=3)
@Priyanshu Mishra
–
I am terrible at geometry, so I would not recommend anything for that. As for functional equations, the key is to do a lot of problems after learning the basic techniques.
I have not found a 'perfect' book, but the good ones are by BJ Venkatachala, Christopher G Small and Problem solving strategies by Arthur Engel.
Actually I am in K.V JNU so i cannot appear in RMO directly. I have to qualify JMO, which i gave this year but unfortunately did not make it. I will try next year( class 11th).
By the way, how was your preparation this year?
Did you study something more this year than what you studied last year for RMO 2014?
@Priyanshu Mishra
–
1 is just a lower bound by noticing that the numerator is greater than the denominator.
For 5, it might help to see that a n tends to infinity, the expression tends to 5
@Siddharth G
–
I saw that but no such problem is there. On second page of polynomials there are points numbered 4, 5, 6 but i am unable to find. Can you tell the point number?
@Priyanshu Mishra
–
The whole concept might be a bit difficult without a grasp of differentiation. But you can still solve the problem, if you know how to differentiate a polynomial.
@Priyanshu Mishra
–
Indeed. I made a mistake in my calculations. This is the system you got right?
a+b+c+d+e+f=1−a+b−c+d−e+f=−15a+4b+3c+2d+e=05a−4b+3c−2d+e=020a+12b+6c+2d=0−20a+12b−6c+2d=0
@Priyanshu Mishra
–
The question is terribly framed. I am not sure but I think it means the AM of two different primes is 27. Fins thw maximal possible value of one of the primes.
@Priyanshu Mishra
–
http://web.geogebra.org/. Though it is preferable that you you should at least try to draw it on your own first.
PS Sorry for the late response. Couldnt solve your previous problem. Do you have a solution?
@Siddharth G
–
I have drawn a number of diagrams in geogebra but could not save it on PC so that I can post problems in BRILLIANT. This is my problem. Please help me that how can I save it in PC.
No I don't have solution to that inequality .
Please tell me how to save the diagrams of geogebra in desktop as soon as possible.
@Priyanshu Mishra
–
I dont think so. On the top right corner, (its like an equivalence sign with horizontal bars), go to export and select png. this works on the web version. An alternate method is to take a screenshot with web apps like Nimbus.
IIT professors don't take it initiative. They are just regional coordinators for name.
In KV, 20 DAYS camp is conducted but rarely a student is selected in INMO, where students from delhi region attend no camp, yet so many qualify. GREAT IRONY!!
Also thanks for helping me to post geometric figures. The method worked.
How was your INMO? Can you send me the paper?
Also, clear my doubts of 2 questions i have posted - "Help residues mod 7 continents".
@Priyanshu Mishra
–
http://services.artofproblemsolving.com/download.php?id=YXR0YWNobWVudHMvYS85LzlkNDNhMWMzNGM3ZDdmNWQ4ZGE3ZDI2NjIxYzg2MThkNjQ2MWM1LmpwZw==&rn=U2Nhbl8yMDE2MDExNy5qcGc=
It was okay.
Asfor the camp, last time we had one professional olympiad teacher (he gave the notes), one prof from IIT and the rest were past INMO awardees, who studied in IITD. This time the problem(I think) was mainly due to lack of time.
@Siddharth G
–
Suppose i have opened NIMBUS on one side and on the webpage i have opened the geogebra, where i have drawn the image. Then how can i take the snapshot of the image by nimbus?
@Siddharth G
–
I qualified JMO and KVS RMO this year and appeared for INMO via KVS region, so I met them at INMOTC and also after the exam, I asked some, who I know can qualify.
@Priyanshu Mishra
–
As per the notes are concerned, I am not that good at taking down notes. However, I can send you the two KVS RMO papers within this week if you give me your e-mail ID. I will send after 2 or 3 days coz we are having ASL tomorrow and the day after. BTW, which class are you in? And did you appear for JMO this year?
@Priyanshu Mishra
–
How did you get to know your marks?? The excel sheet with everyone's marks is not yet out, only those who qualified have got to know their marks.
@Priyanshu Mishra
–
No, I was not asking mine. I was just telling as ur name is not there, how did you come to know your marks?? One friend of mine also rang up Lawania sir, but he didn't tell the marks!!
@Raushan Sharma
–
Actually one maths teacher of my school requested him to tell the my marks of JMO. As he checked my JMO answer sheet, he knew the marks and told orally. He was also impressed by my solutions. He checked my copy 5 times so that i may be selected for INMOTC but my fate was not that good.
@Priyanshu Mishra
–
For integers, (3,2) is the only solution. Just rationalize the numerator and the denominator, and then note that the radical part of each term of the numerator should be 6. So, fix a=2l2 and b=2k2 and then simplify to get (1+l)(1−k)=0. And hence you get the only solution as (3,2)
@Priyanshu Mishra
–
However, if it is asked to find solutions not only in integers but also in rational numbers, then we can generalize a solution to get infinitely many solutions
Easy Math Editor
This discussion board is a place to discuss our Daily Challenges and the math and science related to those challenges. Explanations are more than just a solution — they should explain the steps and thinking strategies that you used to obtain the solution. Comments should further the discussion of math and science.
When posting on Brilliant:
*italics*
or_italics_
**bold**
or__bold__
paragraph 1
paragraph 2
[example link](https://brilliant.org)
> This is a quote
\(
...\)
or\[
...\]
to ensure proper formatting.2 \times 3
2^{34}
a_{i-1}
\frac{2}{3}
\sqrt{2}
\sum_{i=1}^3
\sin \theta
\boxed{123}
Comments
@Siddharth G :- many congrats for clearing RMO!!!. How many problems did you solve?
Log in to reply
Thanks! I attempted 5 questions, expected to get 4-4.5. How was your paper? (GMO right?)
Log in to reply
Read This. I should say, I didn't do as well as you did. (Regardless of the paper's standard)
Nice solution!
The second last line should be; "RHS is maximised when √a+ √b+ √c is 'minimised' " as the whole fraction is maximised when its denominator is minimised.
However your deduction is very perfect.
In which class presently you are?
Log in to reply
11th As for the second last line, a+b+c when maximized, yields 3−(a+b+c) to be minimized,which implies that the RHS is maximized.
Log in to reply
Thanks. Now i understood that.
I am also preparing for RMO. So as an elder please guide me that which book will be helpful for me to clear RMO?
Also i get to know that you have cleared RMO , so please tell me which book helped you a lot.
Log in to reply
Log in to reply
Are 'Problem primer for the olympiads' and 'Number theory-problems, structures and examples'(by titu .A.) not fine for RMO/INMO ?
Log in to reply
Log in to reply
Please help me in solving this question ( KVS JMO 2015)
A polynomial f(x) with rational coefficients leaves remainder 15, when divided by x−3 and remainder 2x+1, when divided by (x−1)2. Find the remainder when f(x) is divided by (x−3)(x−1)2.
Log in to reply
2x2−2x+3. Though I havent found a good solution yet
The answer seems to beLog in to reply
You can tell me your approach, irrespective of good or bad.
Log in to reply
2x+1 when divided by (x−1)2 ,
I took a cubic which left a remainder2x3−4x2+4x+1=(x−1)2(x+2)+2x+1. It left a remainder 31 when divided by x−3. Note that (x−1)2 leaves a remainder of 4 when divided by x−3.
Thus, 2x3−4x2+4x+1−4(x−1)2=2x3−8x2+12x−3 leaves a remainder of 31−4(4)=15. Hence we find the remainder of 2x3−8x2+12x−3 when divided by (x−1)2(x−3).
Log in to reply
f(1)=3 and f(3)=15.
Got a better solution from my friend- Clearly,Then, f(x)=q1(x)(x−3)(x−1)2+r1(x), where r1(x)=ax2+bx+c.
Dividing by (x−1)2 and equating remainders,
2x+1=0+(ax2+bx+c−a(x−1)2) [Multiplied by a to remove the coeff. of x2]
⇒2x+1=(2a−b)x+(c−a) ⇒2=2a−b,1=c−a
Solving with f(1)=3=a+b+c−(1)
(a=2, b=-2, c=3)
Log in to reply
I have your 2nd solution.
What is the problem that i am not able to understand that why you have multiplied x with q1 ( 2nd line) in the polynomial?
You have taken the polynomial as
f(x)=q1(x)(x−3)(x−1)2+r1(x)
I am asking that why you have multiplied x in above polynomial as it is not given?
Log in to reply
q1(x) as a whole is a polynomial and in this case the quotient.(ie q1ofx akin to f(x)
That is not multiplication.Log in to reply
Also give me a clue to solve this problem:
Find all real numbers x for which
10x+11x+12x=13x+14x
Log in to reply
Log in to reply
Please tell me that can i apply CAUCHY - SCHWARZ lemma to prove this inequality ? -
Given a,b,c are positive real numbers such that
a2+b2+c2=3abc.
Prove that:
b2c2a+c2a2b+a2b2c≥a+b+c9
Log in to reply
Log in to reply
a3b2c2a4+b3c2a2b4+c3a2b2c4≥a+b+c9
a3b2c2a4+b3c2a2b4+c3a2b2c4≥a2b2c2(a+b+c)(a2+b2+c2)2
≥a2b2c2(a+b+c)9(abc)2
=a+b+c9
As you have cleared RMO, can you tell me which book you preferred specially for Geometry and functional equations?
Log in to reply
Log in to reply
Any professor or you did by yourself or anyone else?
Also without geometry how can one dream for winning RMO or further olympiads? It is the whole and sole .
Log in to reply
Log in to reply
Please help me to factorise this one:
x2+4y2−2xy−2x−4y−8=0
Log in to reply
Log in to reply
Log in to reply
Hello Siddharth G,
Appeared in RMO 2015? How was it? What is your expected score?
Log in to reply
4 Questions. How was yours?
Log in to reply
Actually I am in K.V JNU so i cannot appear in RMO directly. I have to qualify JMO, which i gave this year but unfortunately did not make it. I will try next year( class 11th).
By the way, how was your preparation this year?
Did you study something more this year than what you studied last year for RMO 2014?
Log in to reply
Log in to reply
Log in to reply
Log in to reply
What is the your probability of clearing RMO?
Log in to reply
Log in to reply
As you have qualified RMO 2014, can you tell me that is the INMO camps conducted in delhi or no camp is conducted?
Log in to reply
Log in to reply
Which topics were taught to you?
If possible, can you send me some notes of that camp to my e-mail?
E-mail- [email protected]
Log in to reply
Log in to reply
Have you solved this RMO question in the exam? :
Show that there are infinitely many triples (x,y,z) of integers such that x3+y4=z31.
Log in to reply
x=−a4,y=a3 for some natural a.
Put z=0 andLog in to reply
Before getting this, what steps you did?
Log in to reply
Log in to reply
Log in to reply
Log in to reply
I was doing this question:
Find all positive integers n such that 3n−1+5n−1 divides 3n+5n.
The solution in the book was:
Note that 1<3n−1+5n−13n+5n<5, so we can have only 3n−1+5n−13n+5n∈{2,3,4}
cases, which are easily checked.
Can you explain me why the solution wrote 1<3n−1+5n−13n+5n<5 above ?
I am not understanding how that expression came.
Log in to reply
3n<3n−1⋅5
The numerator is clearly greater than the denominator. For the second part of the inequality note thatLog in to reply
Why it chose 1 and 5 only?
Log in to reply
Log in to reply
Are the RMO results out?
Log in to reply
Log in to reply
Were they geometry ones?
Log in to reply
Log in to reply
Show that there are infinitely many positive real numbers a which are not integers such that a(a−3a) is an integer.
Which method or theorem you applied?
Log in to reply
Log in to reply
I am unable to draw it even after trying 5 times?
Please elaborate the steps.
Log in to reply
Log in to reply
Log in to reply
Log in to reply
Log in to reply
Log in to reply
Hope that you qualify INMO 2016 also, with flying colours.
Log in to reply
Log in to reply
f(x) is a fifth degree polynomial. It is given that f(x)+1 is divisible by (x−1)3 and f(x)−1 is divisible by (x+1)3. Find f(x).
Log in to reply
f(x)=ax5+bx4+cx3+dx2+ex+f. Note that f(1)−1=f(−1)+1=f′(1)=f′(−1)=f′′(1)=f′′(−1)=0. You have six variables and six equations.
LetLog in to reply
f′(1) and f′′(1)?
What is meant byIntegration or differentiation?
Thanks, but I am unable to understand your solution.
Log in to reply
Log in to reply
f′(1) and f′′(1) as f(1)=a+b+c+d+e+f?
How can i differentiateAccording to my knowledge we can differentiate a function only w.r.t some variable , but here are 6 variables.
Could you elaborate the differentiation here?
Log in to reply
Log in to reply
Log in to reply
Log in to reply
Log in to reply
Log in to reply
Log in to reply
a of f(x) has multiplicity m if (x−a)m∣f(x) and (x−a)m+1 does not divide f(x).
A rootLog in to reply
Sorry for your inconvenience.
Log in to reply
Log in to reply
Can you tell me how to start?
Also thanks a lot for sending INMO 2015 notes.
Log in to reply
Log in to reply
a=-3/8)\ \(c=5/4 (e=-15/8)\ and all rest 0 is the correct answer.
ButLog in to reply
a+b+c+d+e+f=1−a+b−c+d−e+f=−15a+4b+3c+2d+e=05a−4b+3c−2d+e=020a+12b+6c+2d=0−20a+12b−6c+2d=0
Indeed. I made a mistake in my calculations. This is the system you got right?Log in to reply
I have found another solution without calculus(working half an hour on it). It is completely algebraic.
Here it is:
Let f(x)=(x−1)3(ax2+bx+c)−1. ...(1)
As (x+1)3∣f(x)−1 we have
f(x)−1=ax5+x4(b−3a)+x3(c−3b+3a)+x2(3b−3c−a)+x(3c−b)−c−2.
and the RHS is divisible by (x+1)3 , so by dividing RHS by (x+1)3
We have the remainder (−38a−6c+18b)x2+(−48a+16b)x−2c−18a+6b−2=0
So, we have three simultaneous equations viz:
38a+6c−18b=0 ; 48a−16b=0 ; 2c+18a−6b+2=0
which on solving together gives a=8−3 ; b=8−9 ; c=−1.
Putting these values in (1) and doing some tedious calculations we get
f(x)=8−3x5+45x3−815x which is the correct answer.
Can you please rate my solution?
Log in to reply
Log in to reply
Log in to reply
Log in to reply
The arithmetic mean of a pair wise distinct prime numbers is 27. Determine the biggest prime among them.
Log in to reply
Log in to reply
Which inequality will you apply to solve this ?:
a, b, c are real numbers such that their sum is 0 an sum of their square is 1. Find the maximum value of a2.b2.c2.
how you drew the image of the rmo question on the website which you sent me?
Please tell the website where I can draw the diagrams.
Log in to reply
Log in to reply
No I don't have solution to that inequality .
Please tell me how to save the diagrams of geogebra in desktop as soon as possible.
Log in to reply
Log in to reply
Should I need to login on the geogebra?
Log in to reply
Log in to reply
By the way how's your preparation for INMO? Was INMOTC conducted at IIT?
If possible, please send me the assignments. You can take as many days you can.
Log in to reply
Log in to reply
IIT professors don't take it initiative. They are just regional coordinators for name.
In KV, 20 DAYS camp is conducted but rarely a student is selected in INMO, where students from delhi region attend no camp, yet so many qualify. GREAT IRONY!!
Also thanks for helping me to post geometric figures. The method worked.
How was your INMO? Can you send me the paper?
Also, clear my doubts of 2 questions i have posted - "Help residues mod 7 continents".
Please post solutions also.
Log in to reply
Log in to reply
Log in to reply
Log in to reply
Log in to reply
Log in to reply
Log in to reply
Log in to reply
Can you tell me what is meant by the test "SMT" conducted in all DPSs? I got to know about it from my fiitjee friend who studies at DPS RKP.
Log in to reply
Log in to reply
Have you solved question 1 of INMO?
Also help me in these questions:
Log in to reply
Log in to reply
Also you share this problem with your friends of school and FiitJee.
Log in to reply
Log in to reply
Log in to reply
Also is it possible for you to send me the notes of INMOTC conducted in Delhi and Hyderabad?
Log in to reply
Log in to reply
[email protected].
Ok , please send the papers on this e-mail id-I am presently in class 10. Yes, i appeared in JMO in 2015 but could not qualify as i scored just 48 marks out of 100.
If you cannot send the notes, then can you atleast send me the assignments provided during the camp?
You can take as many days you want.
Log in to reply
Log in to reply
By the way, which camp was good - DELHI or HYDERABAD?
Log in to reply
Log in to reply
Log in to reply
Log in to reply
Log in to reply
This is the list of JMO 2015 qualifiers:
Log in to reply
Log in to reply
Log in to reply
Log in to reply
By the way, in which class are you?
Log in to reply
Find all positive integers (a,b) such that 3+b2+a is a rational number.
Log in to reply
(3,2) is the only solution. Just rationalize the numerator and the denominator, and then note that the radical part of each term of the numerator should be 6. So, fix a=2l2 and b=2k2 and then simplify to get (1+l)(1−k)=0. And hence you get the only solution as (3,2)
For integers,When will you send the assignments to my mail?
[email protected]
Can you please send me your notes and assignments on -